Can someone Help me?

Can Someone Help Me?

Answers

Answer 1

The Answer: C I did this yesterday

Step-by-step explanation:


Related Questions

What is the value for the expression (-5)(3 + 4) ?

Answers

Answer:

-35

Step-by-step explanation:

Since there are  parentheses, you start with that. 3+4=7

Now, -5 x 7 =-35

Answer:

-35

hope this helps

have a good day :)

Step-by-step explanation:

3+4=7

-5 x 7=-35

Round 27834 to 2 significant figures

Answers

28000 is the answer.

you count two numbers from front and approximate

A sign has 3 identical parallelogram-shaped stripes as shown. Charles must
outline each stripe with reflective tape. Is one roll of 86 inches of tape enough to
finish the job? Explain.

Answers

Answer:

it looks like the length of one of the parts is

Step-by-step explanation:

sqrt(125) from 10^2 +5^2 = sqrt(125) using the pathogen Therom since the 3 lines are identical since they are parallel you multiply sqrt(125) by 3

so 3×sqrt(125)=33.54 inches of tape which is way less then than 86

Solve.
5. The graph of f(x)= x is reflected across the x-axis. The graph is then
translated 11 units up and 7 units to the left. Write the equation of the
transformed function.

Answers

Given:

The function is:

[tex]f(x)=x[/tex]

The graph of this function reflected across the x-axis. The graph is then  translated 11 units up and 7 units to the left.

To find:

The equation of the transformed function.

Solution:

The translation is defined as

[tex]g(x)=kf(x+a)+b[/tex]                .... (i)

Where, k is stretch factor, a is horizontal shift and b is vertical shift.

If 0<k<1, then the graph compressed vertically by factor k and if k>1, then the graph stretch vertically by factor k.

If k<0, then the graph is reflected across the x-axis.

If a>0, then the graph shifts a units left and if a<0, then the graph shifts a units right.

If b>0, then the graph shifts b units up and if b<0, then the graph shifts b units down.

The graph of this function reflected across the x-axis. The graph is then  translated 11 units up and 7 units to the left.  So, [tex]k=-1, b=11, a=7[/tex]. Putting these value in (i), we get

[tex]g(x)=(-1)f(x+7)+11[/tex]

[tex]g(x)=-(x+7)+11[/tex]                     [tex][\because f(x)=x][/tex]

[tex]g(x)=-x-7+11[/tex]

[tex]g(x)=-x+4[/tex]

Therefore, the required function is [tex]g(x)=-x+4[/tex].

What are the zeros of the quadratic function f(x) = 2x2 – 10x – 3?

Answers

Answer:

[tex]x = \frac{ 5 \ + \ \sqrt{31}}{2} \ , \ x = \frac{ 5 \ - \ \sqrt{31}}{2}[/tex]

Step-by-step explanation:

[tex]2x^2 - 10x - 3 = 0 \\\\a = 2 \ , b = - 10 \ , \ c = - 3 \\\\x = \frac{-b^2\ \pm \ \sqrt{b^2 - 4ac}}{2a}\\\\x = \frac{10 \ \pm \sqrt{(-10)^2 - ( 4 \times 2 \times -3)} }{2 \times 2}\\\\x = \frac{10 \ \pm \sqrt{(100 - ( -24 )} }{4}\\\\x = \frac{10 \ \pm \sqrt{(100 + 24 } }{4}\\\\x = \frac{ 10 \ \pm \sqrt{124}}{4}\\\\x = \frac{ 10 \ \pm \sqrt{4 \times 31}}{4}\\\\x = \frac{ 10 \ \pm \sqrt{2^2 \times 31}}{4}\\\\x = \frac{ 10 \ \pm2 \sqrt{31}}{4}\\\\x = \frac{ 5 \ \pm\sqrt{31}}{2}\\\\[/tex]

[tex]x = \frac{ 5 \ + \ \sqrt{31}}{2} \ , \ x = \frac{ 5 \ - \ \sqrt{31}}{2}[/tex]

write the set builder form of
{1, 1/4, 1/9, 1/16, 1/25}​

Answers

here i hope this answer will help you

I’m struggling in finding the nearest cent, May you please help me

Answers

Answer:

$88.48

Step-by-step explanation:

First, we can check the formula for compound interest, which is

[tex]A = P (1 + r)^{n}[/tex] , with A representing the end amount, r representing the interest rate, and n representing the amount of times compounded. We know that the interest rate is 9% (to convert this to a decimal, we can divide it by 100, resulting in 0.09), the end amount is 700, and the amount of times compounded is however many months are in 2 years (there are 12 months per year, and 12 *2 = 24, so 24 times).

Thus, our equation is

[tex]700 = P (1 + 0.09)^{24}\\700 = P (7.911083)\\[/tex]

Divide both sides by 7.911083 to isolate P, and we get

P = 88.48 (rounded to the nearest cent)

Please help me i'm struggling!

Answers

Answer:

c

Step-by-step explanation:

Answer:

x is greater than or equal to 3

So it is the first answer.

CAN SOMEONE HELP ME WITH THIS PLEASE AND THANK YOU.

Answers

Answer: 3 7/12

Step-by-step explanation:

1. Solving the whole number

4-1=3

2. Solving the fraction parts

3/4 - 1/6=

Find the LCM. The LCM is 12 so we rewrite the factions like this:

9/12-2/12=7/12

3. Add.

3+ 7/12= 3 7/12

Hey there!

4 3/4 - 1 1/6

= 19/4 - 7/6

= (19 × 6) - (7 × 4) / 4 × 6

19 × 6 = 114

7 × 4 = 28

= 114 - 28 / 4 × 6

4 × 6 = 24

114 - 28 / 24

114 - 28 = 86

= 86/24

= 3 7/12

Answer: 86/24 OR 3 7/12

Good luck on your assignment and enjoy your day!

~Amphitrite1040:)

What is the least number of apples that can be shared equally among either 6, 10 or 15 children?

Answers

Answer:

If you mean with out splitting up the apple then we could just give one apple to each child so it would look like this

6 children 6 apples

10 children 10 apples

15 children 15 apples

PLEASE HELP IM LITERALLY BEGGING YOU

Amanda pays $115.00 for shoes
that are 20% off at Everything Shoes.
At Best Footwear, the same shoes
are 15% off, which makes them cost
$7.00 less than their pre-sale price at
Everything Shoes.
What was the original cost for the
shoes at Best Footwear?

Answers

Answer:

Step-by-step explanation:

Everything Shoes:

E * 0.8 = 115

E = 115/.8 = $143.75

~~~~~~~~~~~~~~~~

B * 0.15 = 7

B = 7/.15

B = $46.67

hiiii ^-^ need help again with some math ikr math sucks -v-

Answers

Answer:

5a) -8 .     Plan : addition/negative?

5b) -30 .   Plan: subtraction/negative?

Step-by-step explanation:

Im not so sure about what "Plan" means, hopefully you know..

5a)

-15 + 7 = 7 - 15 = -8

5b) -10 +(-20) = -10 - 20 = -30

If my answer is incorrect, pls correct me!

If you like my answer and explanation, mark me as brainliest!

-Chetan K

Answer:

Below

Step-by-step explanation:

So lets start with the first one:

-15+7

We are going to add these together, and will get -8.

Next we have:

-10+(-20)

When there is both a subtraction or negative sign, and a plus sign, the plus sign is removed.

So we can think of this as:

-10-20

And since the -10 is negative, and we are subtracting it, it is going to get more engative:

-10-20=-30

This is the solution answer.

I do not know what the answer to the plan is, i dont understand what plan means.

what is the relationship between vectors u={1,0} and v={0,-3}?

Answers

Answer: Hi, your answer would be  

C) The vectors are orthogonal because the angle between them is 90°.

Whats 8^6 ÷ 8 pls ty :D

Answers

Answer:

[tex] {8}^{6} \div 8 \\ = {8}^{6} \div {8}^{1} \\ = {8}^{(6 - 1)} \\ = {8}^{5} [/tex]

Answer:

[tex]8^{5}[/tex] or 32768

Step-by-step explanation:

When dividing exponents, you subtract the exponent on the top from the one on the bottom:

[tex]8^{6}[/tex]/[tex]8^{1}[/tex] = [tex]8^5[/tex]  because 6-1 = 5

Multiply the rational expressions 2x^2 - 4x - 6 x^2 - 4 ——————- • ——————— x + 2 2x^2 + 2x

Answers

Answer:

C =(x-3)(x-2)/x

or

x²-5x+6/x

Step-by-step explanation:

2x^2 - 4x - 6/x + 2 * x^2 - 4/2x^2 + 2x

Factorizing the given factors

1) 2x^2 - 4x - 6

= 2x^2 - 6x+2x - 6

= 2x(x-3) +2(x-3)

= (x-3)(2x+2)

2) x^2 - 4

= (x+2) (x-2)

3) 2x^2 + 2x

= 2x( x+1)

Putting the factors

(x-3)(2x+2)/x + 2   *  (x+2) (x-2)/2x( x+1)

= (x-3)(2x+2) (x-2)/2x( x+1)

= 2(x-3)(x+1) (x-2)/2x( x+1)

=(x-3)(x-2)/x

Now multiply

(x-3)(x-2) =x²-5x+6

Then the answer becomes  x²-5x+6/x

test the validity of these statement "if my brother stands first in the class, i give him a watch . Either he stood first or I was out of the station I did not give my brother a watch this time . Therefore I was out of station.

Answers

Answer: yeah looks logically correct

Step-by-step explanation: I he got the watch he was 1st

Fact is he didn't get the watch therefore he wasn't first

Therefore 2nd holds true also - if not first then not in station

No watch = not in station

Is this correct!! Can someone verify!!

Answers

Yeah, it’s totally correct.

hey, this is the last thing I'm asking help with today.thank you

Answers

Answer:

A

Step-by-step explanation:

the middle angle has to be equal to make the entire angle equal

Please help with this question

Answers

Answer:

ans is g=x^3+1

x=(-2-1,0,1,2,3)

again put the value of x

therefore g(x)=(-7,0,1,2,9,28)

Please help me‼️
Question: select all the equations that can be represented by a straight line when graphed on the coordinate plane

Answers

Answer:

B, D, E

Step-by-step explanation:

Straight line equation is in the form of:

ax + by = c

Lets verify the options:

A- no, simplified, it is 1 + xy = 9xB- yesC- no, because of y²D- yesE- yesF - no, simplified it is a quadratic equation

Hallar el menor número no divisible por 4, 6, 9, 11 y 12, tal que al dividirlo por estos números se obtengan restos iguales.

Answers

Answer:

397

Step-by-step explanation:

Sea 'p (x)' el número que no es divisible por 4, 6, 9, 11 y 12 de manera que cuando se divide por estos números da un resto igual, tenemos;

Por teorema del resto, tenemos;

p (x) = (x - a) · Q (x) + R

Dónde;

p (x) = El número, que se divide

Q (x) = El cociente

(x - a) = El divisor

R = El resto

Dado que el número más pequeño es 4, el resto, 0 <R <4

Para el número entero más pequeño arriba (x - a) · Q (x), tenemos;

R = 1

Observamos que el mínimo común múltiplo de 4, 6, 9, 11 y 12 = 396, por lo tanto, podemos tener;

(x - a) · Q (x) = 396

R = 1, dar;

p (x) = 396 + 1 = 397

Por lo tanto;

El número que no es divisible por 4, 6, 9, 11 y 12, de manera que cuando se divide por estos números da residuos iguales, p (x) = 397

Someone help me with this ASAP pls . I need to know “how do we know that <3~=<6

Answers

Answer:

Alternate Interior Angles

Step-by-step explanation:

They are alternate interior angles. The reason this works is because 4 is opposite to 3, meaning that 4 is equivalent to the value of (180-Angle3)

Meanwhile 6 is equivalent to (180-Angle5). Angle 5 is the same value is Angle 4.

help with the question below please!!

Answers

Answer:

a.= 24 b.=-21 c. is x=-7 or 3

Step-by-step explanation:

Step-by-step explanation:

a) f(5)= 5² + 4 x 5 -21

=24

b) f(0)= 0² + 4 x 0 -21

= -21

c) f(x)= 0

=> x² + 4x -21 =0

=> x = 3, x= -7

find the area for this figure

Answers

8 square units

Divide figure into one square, and two triangles

Area of square:
2 x 2 = 4 square units

Area of ONE triangle:
1/2 x 2 x 2 = 2 square units
The triangles we have are both sizes, so multiply this by 2:
2 square units x 2 = 4 square units

ADD THE AREA OF SQUARE AND TRIANGLES:
4 square units + 4 square units = 8 square units

Have a nice day

Please help me.

The table below represents the closing prices of stock ABC for the last five
days. Using your calculator, what is the equation of linear regression that fits
these data?

Answers

Answer:

Step-by-step explanation:

I can tell you how to do this on your calculator, but I have no idea what kind of calculator you have and the methods are different for the various TI graphing calculators, so I'll just tell you that your answer is the last choice given. It's a terrible fit, but if the instructions say linear, the last choice is the equation that the calculator gives.

The regression equation for Y is: ŷ = -0.223X + 20.523

What is Linear equation?

A linear equation is an algebraic equation of the form y=mx+b, where m is the slope and b is the y-intercept, and only a constant and a first-order (linear) term are included. Sometimes, the aforementioned is referred to as a "linear equation of two variables," with y and x serving as the variables.

Sum of X = 15

Sum of Y = 99.27

Mean X = 3

Mean Y = 19.854

The sum of squares (SSX) = 10

The sum of products (SP) = -2.23

Regression Equation = ŷ = bX + a

b = SP/SSₓ = -2.23/10 = -0.223

a = My - bMₓ = 19.85 - (-0.22*3) = 20.523

ŷ = -0.223X + 20.523

Learn more about Linear equations here:

https://brainly.com/question/11897796

#SPJ5

what is the answer? becuase i am just confused

Answers

Answer:

Step-by-step explanation:

radius of circle is 12

also there is 2 right angles in the triangle if you wanted to solve this problem yourself

but here's the answer:

arc angle of the triangle inside the sircle is double the angle closest to x.

(the pointy area of the triangle is half the angle length of the arc)

i can add more though:

NEED ASAP NOWWWWWW TY

Answers

Answer:

subtract 8 from both sides

Step-by-step explanation:

the answer is B im pretty sure

Find the volume of a right circular cone that has a height of 20 ft and a base with a
radius of 9.1 ft. Round your answer to the nearest tenth of a cubic foot.

Answers

Your answer will be 1733.489333ft³
When rounding it will be
1733.1ft³
My rounding might be wrong
Hope it helps : )

Hi can someone please help me with this thanks

Answers

Answer: 18

Does this help??

Ryan took a hike on sunday. he hiked uphill for 15 minutes. then he hiked another River for 12 minutes. he hiked downhill for 8 minutes along a river for 5 minutes. how many seconds did Ryan hike on Sunday? plz help​

Answers

Answer:

2400 seconds

Step-by-step explanation:

Answer:

40 min = 2400 sec

Step-by-step explanation:

Other Questions
All countries have mixed economies. Identify 2 ways the US exhibits elements of a Market Economy and 2 ways the US exhibits elements of a Command Economy. Explain your answer in complete sentences. Please answer this: There are 2.54 centimeters in 1 inch. There are 100 centimeters in 1 meter.To the nearest meter, how many meters are in 158 inches? Jewel is often critical of herself and her ability to perform even the most basic tasks. She has tried to change her negative views about herself but self-deprecating thoughts always seem to break through, and her anxiety increases no matter the situation. Jewel's therapist believes that this is the root of her generalized anxiety disorder. Jewel's therapist is MOST likely following the _____ perspective. salma is 1.45 meters tall. at 3p.m she measures the length of a tree's shadow to 29.35 meters. she stands 24.3 meters away from the tree, so that the tip of her shadow meets the tip of the treels shadow. find the height of the tree to the nearest hundreth of a meter. A number that has both a whole number and a fraction is called a(n) O A) improper fraction OB) mixed number OC) whole fraction OD) combined number Which Progressive Era reform do you thinkhad the greatest impact on the American society?Explain. Write your answer in a well-developedparagraph La Edad Media es el periodo que dur un poco ms de 1000 aos; por lo tanto, puedes concluir que fue de..., ayuda,, Scientists found a patch of grass growing in Manyikeni that is the same species as the grass that grows near Great Zimbabwe--and only there. What does this discovery suggest to historians?(1) that Manyikeni was an outpost for trade between Great Zimbabwe and the port at Kilwa(2) that the farmers in Manyikeni learned techniques from farms in Great Zimbabwe(3) that the wind blew from Great Zimbabwe to Manyikeni, carrying grass seed(4) that Arab traders from Egypt stabled their camels in Manyikeni Gary has four times as many points as Frank. Evan has five times as many as Frank. Dave is two times as many as Frank. altogether they have a total of 1,260 points Give an example of a positive slope Cameron is making trail mix. The ratio of peanuts to raisins to walnuts is 3:2:1. If there are 224 raisins, how many total peanuts and walnuts are there? Why would a manager begin the evaluation process by asking the individual to evaluate his or her contributions to the project? explain the contribution of nepal for maintaining international peace and security. PLEASE HELP!!!! WILL GIVE BRAINLIEST!!!!Is my answer correct? Select the points that are solutions to the system of inequalities. Select all that apply A. (-12, 0) B. (1, 2) C. (0, 0) D. (-10, 1) The product of 4 and the depth of the pool please help me I am the first speaker in a debate on the motion which states that " Unemployment is more of a blessing than a curse" i am proposingWill give brainiest The motion of the ball from the time it was thrown upwards was modeled closely by the function,h(t)=-16t2+128t+144, where h(t) represents the height of the ball in feet after t seconds. After how many seconds did the ball reach the ground?please I need help Lifetime Health ch 19 review questions 1-20 its the version with 5 ppl on the cover Pliss you can help me